Difference between revisions of "2012 AMC 8 Problems/Problem 20"

(Video Solution by SpreadTheMathLove)
 
(46 intermediate revisions by 17 users not shown)
Line 1: Line 1:
 +
==Problem==
 
What is the correct ordering of the three numbers <math> \frac{5}{19} </math>, <math> \frac{7}{21} </math>, and <math> \frac{9}{23} </math>, in increasing order?
 
What is the correct ordering of the three numbers <math> \frac{5}{19} </math>, <math> \frac{7}{21} </math>, and <math> \frac{9}{23} </math>, in increasing order?
  
<math> \textbf{(A)}\hspace{.05in}\frac{9}{23}<\frac{7}{21}<\frac{9}{23}\quad\textbf{(B)}\hspace{.05in}\frac{5}{19}<\frac{7}{21}<\frac{9}{23}\quad\textbf{(C)}\hspace{.05in}\frac{9}{23}<\frac{5}{19}<\frac{7}{21} </math>  
+
<math> \textbf{(A)}\hspace{.05in}\frac{9}{23}<\frac{7}{21}<\frac{5}{19}\quad\textbf{(B)}\hspace{.05in}\frac{5}{19}<\frac{7}{21}<\frac{9}{23}\quad\textbf{(C)}\hspace{.05in}\frac{9}{23}<\frac{5}{19}<\frac{7}{21} </math>  
  
 
<math> \textbf{(D)}\hspace{.05in}\frac{5}{19}<\frac{9}{23}<\frac{7}{21}\quad\textbf{(E)}\hspace{.05in}\frac{7}{21}<\frac{5}{19}<\frac{9}{23} </math>
 
<math> \textbf{(D)}\hspace{.05in}\frac{5}{19}<\frac{9}{23}<\frac{7}{21}\quad\textbf{(E)}\hspace{.05in}\frac{7}{21}<\frac{5}{19}<\frac{9}{23} </math>
  
==Solution==
+
==Solution 1==
 
The value of <math> \frac{7}{21} </math> is <math> \frac{1}{3} </math>. Now we give all the fractions a common denominator.
 
The value of <math> \frac{7}{21} </math> is <math> \frac{1}{3} </math>. Now we give all the fractions a common denominator.
  
Line 15: Line 16:
  
 
Ordering the fractions from least to greatest, we find that they are in the order listed. Therefore, our final answer is <math> \boxed{\textbf{(B)}\ \frac{5}{19}<\frac{7}{21}<\frac{9}{23}} </math>.
 
Ordering the fractions from least to greatest, we find that they are in the order listed. Therefore, our final answer is <math> \boxed{\textbf{(B)}\ \frac{5}{19}<\frac{7}{21}<\frac{9}{23}} </math>.
 +
 +
==Solution 2==
 +
Change <math>7/21</math> into <math>1/3</math>;
 +
<cmath>\frac{1}{3}\cdot\frac{5}{5}=\frac{5}{15}</cmath>
 +
<cmath>\frac{5}{15}>\frac{5}{19}</cmath>
 +
<cmath>\frac{7}{21}>\frac{5}{19}</cmath>
 +
And
 +
<cmath>\frac{1}{3}\cdot\frac{9}{9}=\frac{9}{27}</cmath>
 +
<cmath>\frac{9}{27}<\frac{9}{23}</cmath>
 +
<cmath>\frac{7}{21}<\frac{9}{23}</cmath>
 +
Therefore,  our answer is <math> \boxed{\textbf{(B)}\ \frac{5}{19}<\frac{7}{21}<\frac{9}{23}} </math>.
 +
 +
==Video Solution==
 +
https://youtu.be/pU1zjw--K8M ~savannahsolver
  
 
==See Also==
 
==See Also==
 
{{AMC8 box|year=2012|num-b=19|num-a=21}}
 
{{AMC8 box|year=2012|num-b=19|num-a=21}}
 +
{{MAA Notice}}

Latest revision as of 09:12, 16 July 2024

Problem

What is the correct ordering of the three numbers $\frac{5}{19}$, $\frac{7}{21}$, and $\frac{9}{23}$, in increasing order?

$\textbf{(A)}\hspace{.05in}\frac{9}{23}<\frac{7}{21}<\frac{5}{19}\quad\textbf{(B)}\hspace{.05in}\frac{5}{19}<\frac{7}{21}<\frac{9}{23}\quad\textbf{(C)}\hspace{.05in}\frac{9}{23}<\frac{5}{19}<\frac{7}{21}$

$\textbf{(D)}\hspace{.05in}\frac{5}{19}<\frac{9}{23}<\frac{7}{21}\quad\textbf{(E)}\hspace{.05in}\frac{7}{21}<\frac{5}{19}<\frac{9}{23}$

Solution 1

The value of $\frac{7}{21}$ is $\frac{1}{3}$. Now we give all the fractions a common denominator.

$\frac{5}{19} \implies \frac{345}{1311}$

$\frac{1}{3} \implies \frac{437}{1311}$

$\frac{9}{23} \implies \frac{513}{1311}$

Ordering the fractions from least to greatest, we find that they are in the order listed. Therefore, our final answer is $\boxed{\textbf{(B)}\ \frac{5}{19}<\frac{7}{21}<\frac{9}{23}}$.

Solution 2

Change $7/21$ into $1/3$; \[\frac{1}{3}\cdot\frac{5}{5}=\frac{5}{15}\] \[\frac{5}{15}>\frac{5}{19}\] \[\frac{7}{21}>\frac{5}{19}\] And \[\frac{1}{3}\cdot\frac{9}{9}=\frac{9}{27}\] \[\frac{9}{27}<\frac{9}{23}\] \[\frac{7}{21}<\frac{9}{23}\] Therefore, our answer is $\boxed{\textbf{(B)}\ \frac{5}{19}<\frac{7}{21}<\frac{9}{23}}$.

Video Solution

https://youtu.be/pU1zjw--K8M ~savannahsolver

See Also

2012 AMC 8 (ProblemsAnswer KeyResources)
Preceded by
Problem 19
Followed by
Problem 21
1 2 3 4 5 6 7 8 9 10 11 12 13 14 15 16 17 18 19 20 21 22 23 24 25
All AJHSME/AMC 8 Problems and Solutions

The problems on this page are copyrighted by the Mathematical Association of America's American Mathematics Competitions. AMC logo.png